MAT334-2018F > Quiz-7

TUT 0301

<< < (2/2)

Victor Ivrii:
The arguments are correct but two errors, cancelling one another: $\arctan (1/2)\ne \pi /6$ (you could simply write  $\arctan (1/2)$).

Navigation

[0] Message Index

[*] Previous page

Go to full version